Quantcast
  • Register
PhysicsOverflow is a next-generation academic platform for physicists and astronomers, including a community peer review system and a postgraduate-level discussion forum analogous to MathOverflow.

Welcome to PhysicsOverflow! PhysicsOverflow is an open platform for community peer review and graduate-level Physics discussion.

Please help promote PhysicsOverflow ads elsewhere if you like it.

News

PO is now at the Physics Department of Bielefeld University!

New printer friendly PO pages!

Migration to Bielefeld University was successful!

Please vote for this year's PhysicsOverflow ads!

Please do help out in categorising submissions. Submit a paper to PhysicsOverflow!

... see more

Tools for paper authors

Submit paper
Claim Paper Authorship

Tools for SE users

Search User
Reclaim SE Account
Request Account Merger
Nativise imported posts
Claim post (deleted users)
Import SE post

Users whose questions have been imported from Physics Stack Exchange, Theoretical Physics Stack Exchange, or any other Stack Exchange site are kindly requested to reclaim their account and not to register as a new user.

Public \(\beta\) tools

Report a bug with a feature
Request a new functionality
404 page design
Send feedback

Attributions

(propose a free ad)

Site Statistics

205 submissions , 163 unreviewed
5,047 questions , 2,200 unanswered
5,345 answers , 22,709 comments
1,470 users with positive rep
816 active unimported users
More ...

  NOT Universal Operator and Computational basis

+ 1 like - 1 dislike
1245 views

This is the relationship between density operator and Bloch vector: $$\rho= \frac{1}{2}({\bf{\hat{1}}}+{\bf{b}}.\boldsymbol{\hat{\sigma}})$$

We define the NOT Universal Operator in the following way:

$$U: {\bf{b}}\to -{\bf{b}}/3$$

My question is - How does NOT Universal Operator act on the elements of the computational basis: $|0\rangle, |1\rangle$?

This post imported from StackExchange Physics at 2014-06-11 15:05 (UCT), posted by SE-user user15940
asked Jan 1, 2013 in Theoretical Physics by user15940 (0 points) [ no revision ]
retagged Jun 11, 2014
Related physics.stackexchange.com/questions/26864/…

This post imported from StackExchange Physics at 2014-06-11 15:05 (UCT), posted by SE-user twistor59
I put some bold symbols in your question because the quantities involved are vectors. Can I ask you to confirm 1) the bold is OK, and that 2) you really meant to have the /3? I thought UNOT just mapped antipodally, so there should be no -3 there.

This post imported from StackExchange Physics at 2014-06-11 15:05 (UCT), posted by SE-user twistor59
This definition of the Universal NOT seems to differ from the one given in the related question: the OP's (current) definition maps states antipodally but introduces a "damping" of the Bloch vector.

This post imported from StackExchange Physics at 2014-06-11 15:05 (UCT), posted by SE-user Juan Bermejo Vega

Homework, downvoted.

1 Answer

+ 4 like - 0 dislike

Note As it has been said in the comments, this definition of Universal-NOT gate seems to differ from others discussed in other posts [1]. This answer uses the definition proposed by the OP, i.e.

$$\rho=\frac{1}{2}(I+\vec{b}\cdot\vec{\sigma}) \quad \longrightarrow \quad U(\rho)=\frac{1}{2}\left(I-\frac{1}{3}\vec{b}\cdot\vec{\sigma}\right)$$ Where I use the symbol $I$ for the identity matrix to avoid confusion with 1 and $\vec{b}\in \mathbb{R}^3$ denotes the Bloch vector.

We write the density matrices of the computational basis states explicitly: $$ \rho_a=|a\rangle\langle a |= \frac{1}{2}(I+\vec{b}_a\cdot\vec{\sigma}),$$ where $a\in\{0,1\}$. Expanding this expression readily yields the vectors $b_a$: $$\vec{b}_a=(0,0,\pm1).$$ Applying your definition of $U$ to these density operators, the action of the operator on basis states can be obtained directly: $$ U(|0\rangle\langle 0|)= \frac{1}{2}(I- \frac{1}{3}\sigma_z)=\frac{1}{3}|0\rangle\langle 0| +\frac{2}{3}|1\rangle\langle 1 |$$ $$ U(|1\rangle\langle 1|)= \frac{1}{2}(I+ \frac{1}{3}\sigma_z)=\frac{2}{3}|0\rangle\langle 0| +\frac{1}{3}|1\rangle\langle 1 |$$ We can observe that, because the factor $1/3$ that "damps" the Bloch vector, pure basis states evolve into mixed states; notice that, intuitively, states get closer to the totally mixed state $I/2$ if you make the Bloch vector $\vec{b}$ go to zero.

This post imported from StackExchange Physics at 2014-06-11 15:05 (UCT), posted by SE-user Juan Bermejo Vega
answered Jan 1, 2013 by jbvega (285 points) [ no revision ]
+1 Interested to know if the /3 was intentional..

This post imported from StackExchange Physics at 2014-06-11 15:05 (UCT), posted by SE-user twistor59
Thank you very much for your answer.

This post imported from StackExchange Physics at 2014-06-11 15:05 (UCT), posted by SE-user user15940
You are welcome. If my reply answers all your questions then you could accept it. (I would normally not say this, but since there has not been any activity in your post for a very long time I guess your original problem has been solved.)

This post imported from StackExchange Physics at 2014-06-11 15:05 (UCT), posted by SE-user Juan Bermejo Vega
I mean, this is one of sites policies that you should keep not only for this question, but also whenever you ask something in physics.SE: it is nice to accept some answer that you like, find useful or solves completely (or partially) your original problem. If you are not satisfied with the answers you should try to say why to the people who try to help you.

This post imported from StackExchange Physics at 2014-06-11 15:05 (UCT), posted by SE-user Juan Bermejo Vega

Your answer

Please use answers only to (at least partly) answer questions. To comment, discuss, or ask for clarification, leave a comment instead.
To mask links under text, please type your text, highlight it, and click the "link" button. You can then enter your link URL.
Please consult the FAQ for as to how to format your post.
This is the answer box; if you want to write a comment instead, please use the 'add comment' button.
Live preview (may slow down editor)   Preview
Your name to display (optional):
Privacy: Your email address will only be used for sending these notifications.
Anti-spam verification:
If you are a human please identify the position of the character covered by the symbol $\varnothing$ in the following word:
p$\hbar$ysicsOverflo$\varnothing$
Then drag the red bullet below over the corresponding character of our banner. When you drop it there, the bullet changes to green (on slow internet connections after a few seconds).
Please complete the anti-spam verification




user contributions licensed under cc by-sa 3.0 with attribution required

Your rights
...